Đến nội dung

xuanhoan23112002 nội dung

Có 95 mục bởi xuanhoan23112002 (Tìm giới hạn từ 29-04-2020)



Sắp theo                Sắp xếp  

#709350 Đề thi vào 10 chuyên Lê Hồng Phong - Nam Định 2018 - 2019 - Toán Chuyên

Đã gửi bởi xuanhoan23112002 on 27-05-2018 - 10:49 trong Tài liệu - Đề thi

Câu 5:

a. Có $\frac{1}{2}\sqrt{(a+3b)(b+3a)}\leq \frac{a+3b+b+3a}{4}=a+b$ (bất đẳng thức AM-GM)

Từ giả thiết: $\sqrt{a}+\sqrt{b}=1$ 

Bình phương 2 vế ta có: $2\sqrt{ab}=1-a-b$ 

Hay $4ab=(1-a-b)^2$

Nên bất đẳng thức cần chứng minh tương đương với:

$3(a+b)^2+(1-a-b)^2\geq 2(a+b)$

$\Leftrightarrow (2a+2b-1)^2\geq 0$ (luôn đúng)

Vậy bất đẳng thức đã cho được chứng minh.




#712416 Đề thi IMO 2018

Đã gửi bởi xuanhoan23112002 on 12-07-2018 - 21:52 trong Thi HSG Quốc gia và Quốc tế

Kết quả chính thức IMO 2018



#705302 ĐỀ THI OLYMPIC 30/4 NĂM 2018 THPT LHP TP.HCM - KHỐI 10

Đã gửi bởi xuanhoan23112002 on 08-04-2018 - 23:13 trong Thi HSG cấp Tỉnh, Thành phố. Olympic 30-4. Đề thi và kiểm tra đội tuyển các cấp.

Bài 4:

Theo đề bài ta có: $p^2 -p+1=x^3$(x là số tự nhiên,x>1)

Hay $p(p-1)=(x-1)(x^2+x+1)$

Do $p$ là số nguyên tố nên $x-1$ hoặc $x^2+x+1$ chia hết cho$ p$

Nếu $x-1$ chia hết cho $p$ thì $x-1\ge p,x^2+x+1<p$ (vô lí với x là số tự nhiên >1) 

Do đó $x^2+x+1$ chia hết cho $p$ nên $x^2+x+1=pk$ (k là số tự nhiên)

Ta xét$ k=1,2$ không thỏa mãn

Xét $k\ge 3$

Thay vào phương trình ta được:$ p-1=(x-1)k$ hay $p=(x-1)k+1$

Từ đó ta có: $x^2+x+1=(xk-k+1)k$

Hay $x^2+x(1-k^2)+k^2-k+1=0$

Đây là phương trình bậc 2 ẩn x, để phương trình có nghiệm tự nhiên thì $\delta =k^4-6k^2+4k-3$ phải là số chính phương

Ta có: $(k^2-3)^2\le k^4-6k^2+4k-3<(k^2-2)^2$

Từ đó ta tìm được k=3 ta tìm được x=7, p=19 là số nguyên tố

Vậy p=19 là số nguyên tố thỏa mãn đề bài




#709395 Đề thi vào trường chuyên Thái Bình năm 2019 (vòng 2)

Đã gửi bởi xuanhoan23112002 on 27-05-2018 - 20:34 trong Tài liệu - Đề thi

Câu 6:

Ta có:$\frac{1}{\sqrt{5a^2+2ab+2b^2}}=\frac{1}{\sqrt{(2a+b)^2+(a-b)^2}}\leq \frac{1}{2a+b}\leq \frac{1}{9}(\frac{2}{a}+\frac{1}{b})$ (bất đẳng thức Schwarz)

Chứng minh tương tự như trên ta có:

$P\leq \frac{1}{3a}+\frac{1}{3b}+\frac{1}{3c}$

Ta cũng có:$\frac{1}{a}+\frac{1}{b}+\frac{1}{c}\leq \sqrt{3(\frac{1}{a^2}+\frac{1}{b^2}+\frac{1}{c^2})}=\sqrt{3}$ (bất đẳng thức AM-GM)

Từ đó ta có: $P\leq \frac{\sqrt{3}}{3}$

Đẳng thức xảy ra $\Leftrightarrow a=b=c=\sqrt{3}$

Vậy MaxP = $\frac{\sqrt{3}}{3}\Leftrightarrow a=b=c=\sqrt{3}$.




#709502 Bất đẳng thức chọn lọc ôn chuyên

Đã gửi bởi xuanhoan23112002 on 29-05-2018 - 15:47 trong Bất đẳng thức và cực trị

Bài 3: Theo giả thiết ta có $0\leq a, b, c\leq 4$ nên

$$(4-a)(4-b)(4-c) \geq 0$$

$\Leftrightarrow 64+4(ab+bc+ca) \geq abc+16(a+b+c)$

$\Leftrightarrow ab+bc+ca\geq 8+\frac{abc}{4}\geq 8$ 

Do đó ta có: $P=(a+b+c)^2-(ab+bc+ca)\leq 36-8=28$

Đẳng thức xảy ra $\Leftrightarrow (a, b, c)=(0, 2, 4)$ và các hoán vị của nó

Vậy $MaxP=28$ $\Leftrightarrow (a, b, c)=(0, 2, 4)$ và các hoán vị của nó




#710076 Đề thi tuyển sinh vào chuyên Lê Quý Đôn tỉnh Quảng Trị năm 2018-2019

Đã gửi bởi xuanhoan23112002 on 06-06-2018 - 07:55 trong Tài liệu - Đề thi

Câu 6:

Từ giả thiết kết hợp với công thức khai triển bậc 4: $(a+b)^4=a^4+4a^3b+6a^2b^2+4ab^3+b^4$ ta có:

$Q=(x+3-x)^4-4x(3-x)(x^2+(3-x)^2)=81-2(9-x^2-(3-x)^2)(x^2+(3-x)^2)=81+2(x^2+(3-x)^2)^2-18(x^2+(3-x)^2)=2(x^2+(3-x)^2-5)^2+2(x^2+(3-x)^2)+31\geq 10+31=41$

Đẳng thức xảy ra  $$\Leftrightarrow x^2+(3-x)^2=5\Leftrightarrow 2x^2-6x+4=0\Leftrightarrow x=1, x=2$$

Vậy $Min Q=41\Leftrightarrow x=1, x=2$




#709874 $a^2 + b^2 + c^2 + abc = 4$

Đã gửi bởi xuanhoan23112002 on 04-06-2018 - 07:45 trong Bất đẳng thức và cực trị

Bạn có thể tham khảo tại đây: http://diendantoanho...2b2c2abc-geq-4/

Đáp án: $Min P=2\Leftrightarrow (a,b,c)=(2,0,0)$ và các hoán vị của nó

             $Max P=3\Leftrightarrow (a,b,c)=(1,1,1)$




#709905 $a^2 + b^2 + c^2 + abc = 4$

Đã gửi bởi xuanhoan23112002 on 04-06-2018 - 12:38 trong Bất đẳng thức và cực trị

Tìm max: http://diendantoanho...-định-năm-2018/ (chỉ việc thay mỗi số 2 thành số 1 thôi a trình bày đầy đủ rồi)

Tìm min:

Nếu cả 3 số a, b, c đều > 2 hiển nhiên suy ra điều vô lí

Do đó ta giả sử: $c\leq 2$ nên $abc\leq 2ab$

$\Rightarrow 4=a^2+b^2+c^2+2abc\leq a^2+b^2+c^2+2ab=(a+b)^2+c^2\leq (a+b+c)^2$

$\Rightarrow a+b+c\geq 2$




#710107 Đề thi tuyển sinh vào lớp 10 chuyên toán tỉnh Kiên Giang

Đã gửi bởi xuanhoan23112002 on 06-06-2018 - 14:53 trong Tài liệu - Đề thi

Bài 7: Bài bất đẳng thức có vẻ dễ nhỉ

Ta có:

$\frac{x^2}{y+z}+\frac{y^2}{z+x}+\frac{z^2}{x+y}\geq \frac{(x+y+z)^2}{2(x+y+z)}= \frac{x+y+z}{2}=1$ ( bất đẳng thức Schwarz)

Đẳng thức xảy ra $\Leftrightarrow x=y=z=\frac{2}{3}$

Vậy ta có điều phải chứng minh.




#669686 Trong 100.000.000 số hạng đầu tiên của dãy Fibonacci, có tồn tại hay...

Đã gửi bởi xuanhoan23112002 on 24-01-2017 - 13:57 trong Tổ hợp và rời rạc

Ký hiệu $F_{n}$ là số hạng thứ $n$ của dãy Fibonacci

Chia mỗi số $F_{n}$ cho $10^4$ đc các số dư là $r_{n}$ (mod $10^4$) với $r_{i}=\overline{0,9999}$

Xét tất cả các cặp có thể có của các số nguyên từ $0$ đến $9999$ có $10^8$ cặp $(r_{0};r_{0});(r_{0};r_{1});(r_{1};r_{2});...$

Giả sử ko có cặp nào là $(0;0)$ thì các cặp trên phải có 2 cặp trùng nhau theo nguyên lý Dirichlet, gsử:

  $r_{n}=r_{p}$ và $r_{n+1}=r_{p+1}$ ($n>p$)

$\Rightarrow r_{n-1}=r_{p-1} \Rightarrow r_{n-2}=r_{p-2} \Rightarrow ... \Rightarrow r_{1}=r_{n-p+2}=r_{n-p+1} \Rightarrow r_{n-p+1}=r_{n-p+2} \Rightarrow r_{n-p}=0$

Vậy tồn tại $1$ số $F_{n}$ chia hết cho $10^4$ hay có tận cùng là $4$ chữ số $0$

Mình không hiểu bài này lắm, gửi lại cho mình đi.




#710048 Đề thi tuyển sinh vào lớp 10 THPT chuyên tỉnh Vĩnh Phúc năm học 2018-2019

Đã gửi bởi xuanhoan23112002 on 05-06-2018 - 19:45 trong Tài liệu - Đề thi

Câu 5:

Từ giả thiết ta có: $c=a+b-\sqrt{ab}$

$P=\frac{c^2}{ab}+\frac{c^2}{a^2+b^2}+\frac{\sqrt{ab}}{a+b}$

$P\geq c^2(\frac{1}{2ab}+\frac{1}{2ab}+\frac{1}{a^2+b^2})+\frac{\sqrt{ab}}{a+b}\geq \frac{9c^2}{(a+b)^2+2ab}+\frac{\sqrt{ab}}{a+b} \geq \frac{6(a+b-\sqrt{ab})^2}{(a+b)^2}+\frac{\sqrt{ab}}{a+b}=6-\frac{11\sqrt{ab}}{a+b}+\frac{6ab}{(a+b)^2}=6(\frac{\sqrt{ab}}{a+b}-\frac{1}{2})^2-\frac{5\sqrt{ab}}{a+b}+\frac{9}{2}$ $\geq -\frac{5}{2}+\frac{9}{2}=2$ ( theo các BĐT AM-GM và Schwarz)

 Đẳng thức xảy ra $\Leftrightarrow a=b=c>0$

Vậy $MinP=2\Leftrightarrow a=b=c>0$




#710186 Đề thi tuyển sinh vào lớp 10 THPT chuyên tỉnh Vĩnh Phúc năm học 2018-2019

Đã gửi bởi xuanhoan23112002 on 07-06-2018 - 11:41 trong Tài liệu - Đề thi

Câu 3 (phỏng theo lời giải của thầy Võ Quốc Bá Cẩn) 

Ta có: $\frac{a^2}{a^2+ab+b^2}+\frac{c^2}{c(a+b+c)}\geq \frac{(a+c)^2}{a^2+b^2+c^2+ab+bc+ca}$ (bất đẳng thức Schwarz)

Làm tương tự với 3 phân thức còn lại ta có:

$\frac{a^2}{a^2+ab+b^2}+\frac{b^2}{b^2+bc+c^2}+\frac{c^2}{c^2+ca+a^2}\geq 1$

Đẳng thức xảy ra $\Leftrightarrow a=b=c> 0$

Vậy ta có điều phải chứng minh.




#691945 $P=\frac{(x+1)^{2}(y+1)^{2}}{(x-...

Đã gửi bởi xuanhoan23112002 on 31-08-2017 - 13:28 trong Bất đẳng thức và cực trị

Bài này khá dễ giống đề thi lớp 10 ở VĨnh Phúc,. Bạn có thể tham khảo, hãy like cho mình nhé




#709806 Tìm GTLN của $2x^2-3xy-2y^2$

Đã gửi bởi xuanhoan23112002 on 03-06-2018 - 07:30 trong Bất đẳng thức và cực trị

Ta có:

$A-36a=(2-25a)x^2-(3+20a)xy-(2-40a)y^2$

Coi phương trình trên là phương trình bậc 2 ẩn x tìm giá trị của a sao cho phương trình có nghiệm kép tức là$\Delta =0$ (Chú ý: Tìm giá trị lớn nhất thì $A-36a$ mang dấu trừ của 1 bình phương đủ nên $2-25a<0, 2-40a<0$)

Từ đó tìm được: $a=\frac{1}{12 }$




#709817 Đề thi lớp 10 môn Toán vào Trường THPT Chuyên Lam Sơn

Đã gửi bởi xuanhoan23112002 on 03-06-2018 - 10:57 trong Tài liệu - Đề thi

Câu 5: Bất đẳng thức đã cho tương đương với:

$a^2b+b^2c+c^2a+ab^2+bc^2+ca^2> a^3+b^3+c^3+2abc$

$\Leftrightarrow a(b-c)^2+b^2(c+a-b)+c^2(a+b-c)> 0$ (luôn đúng đó a, b, c là độ dài 3 cạnh của 1 tam giác)




#709371 CMR: $\sum \frac{ab+c^{2}}{a+b}...

Đã gửi bởi xuanhoan23112002 on 27-05-2018 - 17:05 trong Bất đẳng thức và cực trị

Ta có: $\sum \frac{ab+c^2}{a+b}+\sum c= \sum \frac{(c+a)(c+b)}{a+b}\geq 2(a+b+c)$ (bất đẳng thức AM-GM)

$\Rightarrow \sum \frac{ab+c^2}{a+b}\geq a+b+c$

Đẳng thức xảy ra $\Leftrightarrow a=b=c> 0$

Vậy bất đẳng thức được chứng minh.




#705140 chứng minh

Đã gửi bởi xuanhoan23112002 on 06-04-2018 - 23:36 trong Số học

Lời giải nhanh nhất cho bài toán này là cách áp dụng định lý Fermat nhỏ:

x- x chia hết cho 5

Dễ dàng chứng minh x5 - x chia hết cho 2

Như vậy ta cũng có điều phải chứng minh




#704021 2(ab+bc+ca)+$\frac{1}{ab}+\frac{1...

Đã gửi bởi xuanhoan23112002 on 20-03-2018 - 22:08 trong Bất đẳng thức và cực trị

bạn bị ngược dấu hay sao ấy

nguoc dau cho nao




#703960 2(ab+bc+ca)+$\frac{1}{ab}+\frac{1...

Đã gửi bởi xuanhoan23112002 on 19-03-2018 - 23:21 trong Bất đẳng thức và cực trị

Su dung pp pqr
Dat p=a+b+c=3

q=ab+bc+ca

r=abc,r<=1

BDT tuong duong 2q+3/r>=9

Hay 2qr+3>=9r

Ma q>=3*can(r)( do q^2>=3pr)

Dua ve bpt an r giai voi chu y r<=1




#705303 Cần lắm một lời giải thích !

Đã gửi bởi xuanhoan23112002 on 08-04-2018 - 23:22 trong Hình học phẳng

Đó là tính chất góc định hướng được tạo bởi 4 điểm đồng viên




#705345 Cần lắm một lời giải thích !

Đã gửi bởi xuanhoan23112002 on 09-04-2018 - 20:48 trong Hình học phẳng

Ý mk hỏi [mod $\pi$] nghĩa là gì

góc định hướng giữa 2 đường thẳng hơn nhau 1 bội của π

 




#709300 Bất đẳng thức trong đề thi vào lớp 10 Nam Định năm 2018

Đã gửi bởi xuanhoan23112002 on 26-05-2018 - 17:55 trong Bất đẳng thức và cực trị

Đây là đề của ban xã hội bạn ạ.

Còn đây là lời giải của mình cho bài toán này các bạn có thể tham khảo:

Coi phương trình trên là phương trình bậc 2 ẩn a theo công thức nghiệm ta được

$a=\frac{-bc+\sqrt{(4-b^2)(4-c^2)}}{2}$

Áp dụng bất đẳng thức Cauchy cho căn thức trong biểu thức trên, ta có:

$a\leq \frac{-bc+\frac{4-b^2+4-c^2}{2}}{2}= \frac{8-(b+c)^2}{4}$

Từ đó ta có: $2a+b+c=\frac{8-(b+c)^2+2(b+c)}{2}=\frac{9-(b+c-1)^2}{2}\leq \frac{9}{2}$

P/s: Mình nghĩ đây là cách ngắn nhất và có thể thay số 2 trong đề bài bởi các số khác vẫn có thể giải tương tự.




#709289 Bất đẳng thức trong đề thi vào lớp 10 Nam Định năm 2018

Đã gửi bởi xuanhoan23112002 on 26-05-2018 - 15:38 trong Bất đẳng thức và cực trị

Cho a, b, c là các số thực dương thỏa mãn: $a^2+b^2+c^2+abc=4$. Chứng minh rằng:

$2a+b+c\leq \frac{9}{2}$




#706210 Chứng minh MB vuông góc MN khó

Đã gửi bởi xuanhoan23112002 on 17-04-2018 - 20:40 trong Hình học

Bài này có thể sử dụng tích vô hướng của lớp 10




#709756 Tìm GTLN của $2x^2-3xy-2y^2$

Đã gửi bởi xuanhoan23112002 on 02-06-2018 - 07:23 trong Bất đẳng thức và cực trị

Đặt $A=2x^2-3xy-2y^2$

$\Leftrightarrow$$A-3=2x^2-3xy-2y^2-\frac{1}{12}(25x^2-20xy+40y^2)$

$\Leftrightarrow$$A-3=-\frac{1}{12}x^2-\frac{4}{3}xy-\frac{16}{3}y^2$

$\Leftrightarrow$$A-3=-\frac{1}{12}(x+8y)^2\leq 0$

$\Leftrightarrow A\leq 3$

Đẳng thức xảy ra $\Leftrightarrow (x, y)=(\frac{4\sqrt{2}}{5}, -\frac{\sqrt{2}}{10})$ hoặc $(x, y)=(-\frac{4\sqrt{2}}{5}, \frac{\sqrt{2}}{10})$

Vậy Max của $2x^2-3xy-2y^2=3$